Polar Coordinates: Arc length of two overlapping curves

Click For Summary
The discussion focuses on finding the perimeter inside the curve r=15sin(theta) and outside r=1, specifically determining the angle where these curves intersect. The user struggles with expressing the angle in terms of pi, as they need to find theta for which sin(theta) equals 1/15. It is noted that the inverse sine function cannot generally be expressed as rational multiples of pi, as these are transcendental functions. However, it is mentioned that in this specific problem, the appearance of sin^{-1}(1/15) cancels out in the final result, simplifying the calculation. Overall, the conversation highlights the challenges of working with transcendental functions in polar coordinates.
Badgerspin
Messages
15
Reaction score
0
This question may be something of a dumb one. I feel I should know this, but well, I don't.

I'm being asked to find the perimeter inside of the curve r=15sin(theta) and outside of r = 1

Setting up the equation I can do. If it were just an indefinite integral, this would be cake. My challenge right now is finding the angle in which to compute the problem. From where to where? Let x = theta

15sin(x) = 1
Sin(x) = 1/15

For what value of theta would I get 1/15? I can get the numeric value by taking the arcsin, but I need to be able to show it in the format (pi/#, or perhaps ((#pi)/#).

While I'm on that note, for future reference, is there any easy way to compute something like this where I'm being asked oddball angles?
 
Physics news on Phys.org
Badgerspin said:
For what value of theta would I get 1/15? I can get the numeric value by taking the arcsin, but I need to be able to show it in the format (pi/#, or perhaps ((#pi)/#).

While I'm on that note, for future reference, is there any easy way to compute something like this where I'm being asked oddball angles?

In general you cannot express \sin^{-1}x in terms of rational multiples of \pi. You can't express \sin \phi in elementary terms either for general \phi either. That why these functions are referred to by the term transcendental.

In this particular problem, it looks like the appearances of \sin^{-1}(1/15) cancel out in the final result, so you never have to worry about it.
 
Question: A clock's minute hand has length 4 and its hour hand has length 3. What is the distance between the tips at the moment when it is increasing most rapidly?(Putnam Exam Question) Answer: Making assumption that both the hands moves at constant angular velocities, the answer is ## \sqrt{7} .## But don't you think this assumption is somewhat doubtful and wrong?

Similar threads

Replies
7
Views
2K
Replies
2
Views
2K
  • · Replies 5 ·
Replies
5
Views
2K
  • · Replies 19 ·
Replies
19
Views
2K
  • · Replies 1 ·
Replies
1
Views
2K
  • · Replies 2 ·
Replies
2
Views
3K
  • · Replies 22 ·
Replies
22
Views
2K
  • · Replies 3 ·
Replies
3
Views
3K
  • · Replies 24 ·
Replies
24
Views
3K
  • · Replies 2 ·
Replies
2
Views
2K